Docsity
Docsity

Prepare for your exams
Prepare for your exams

Study with the several resources on Docsity


Earn points to download
Earn points to download

Earn points by helping other students or get them with a premium plan


Guidelines and tips
Guidelines and tips

NCLEX Questions and Answers, Exams of Nursing

Over 400 questions and answers related to nursing and healthcare. The questions cover a wide range of topics, including patient care, medication administration, and disease management. The answers provide detailed explanations and rationales for each question, making it a useful study tool for nursing students preparing for the NCLEX exam.

Typology: Exams

2023/2024

Available from 01/19/2024

doksity
doksity 🇺🇸

2K documents

1 / 103

Toggle sidebar

Related documents


Partial preview of the text

Download NCLEX Questions and Answers and more Exams Nursing in PDF only on Docsity! NCSBN NCLEX OVER 400 QUESTIONS & ANSWERS (ASSURED GRADE A PASS) A LPN complains to the charge nurse that an unlicensed assistive person (UAP) consistently leaves the work area untidy and does not restock supplies. What is the best initial response by the charge nurse? 1Write down potential solutions to the problems today by shift's end 2Add this concern to the agenda of the next unit meeting 3Assure the staff nurse that the complaint will be investigated 4Explore for further identification about the nature of the problem - Correct answer 4 Explore for further identification about the nature of the problem The nurse assists with the reinforcement of information about breast self-examination to a group of college students. A female student asks when to perform the monthly exam. The appropriate reply by the nurse should include which statement? 1"Ovulation, or bicycle is the best time to detect changes." 2"Do the exam at the same time every month." 3"Right after the period, when your breasts are less tender." 4"The first of every month, because it will be easiest to remember." - Correct answer 3 The nurse is caring for a 75 year-old client with type 2 diabetes mellitus. The client should be instructed to contact the outpatient clinic immediately if which findings are present? 1An open wound on the heel with minimal discomfort 2Occasional hiccups and sneezing 3Sustained insomnia and daytime fatigue 4Persistent dryness and itching of the perinea area - Correct answer 1An open wound on the heel with minimal discomfort- A pregnant woman has been advised to alter her diet during pregnancy by increasing the intake of protein and vitamin C to meet the needs of the growing fetus. Which diet choice would best meet the woman's needs? 1. 1 cup of macaroni, three-fourths cup of peas, glass whole milk, medium pear 2. Scrambled egg, hash browned potatoes, one-half glass of buttermilk, large nectarine 3. 3 oz. chicken, one-half cup of corn, lettuce salad, small banana 4. Beef, one-half cup of lima beans, glass of skim milk, three-fourths cup of strawberries - Correct answer 4. Beef, one-half cup of lima beans, glass of skim milk, three-fourths cup of strawberries - A nurse is taking a health history from parents of a child admitted with possible Reye's syndrome. Which recent illness should the nurse recognize as being associated with an increased the risk for the development of Reye's syndrome? 1. Varicella 2. Meningitis 3. Hepatitis 4. Rubella - Correct answer 1. Varicella - A Native American chief visits his newborn son and performs a traditional ceremony that involves feathers and chanting. The nurse comments to a colleague: "I wonder if he has any idea how ridiculous he looks - he's a grown man!" The nurse's comment is an example of what type of attitude? 1. Prejudice 2. Ethnocentrism 3. Discrimination 4. Stereotyping - Correct answer 1. Prejudice- A nursing student asks the licensed practical nurse (LPN) to explain the forces that drive health care reform. When responding to the student's question, what information should the nurse emphasize? 1. Increased competition between health care insurers 2. Increase in health care spending that's growing faster than the economy 3. Increase in the population who have health insurance 4. Increase in spending for end-of-life treatment - Correct answer 2 A child is admitted to the unit with the suspected diagnosis of pertussis (whooping cough). What is the priority nursing intervention for this child? 1. Maintain hydration and encourage fluids 2. Implement droplet precautions 3. Monitor respiratory rate and oxygen saturation 1Less jaundice 2Increased appetite 3Decreased lethargy 4Less edema - Correct answer 3 The LPN is unsure about an assignment by the charge nurse to hang an intravenous (IV) infusion that contains potassium. What resource should the LPN check first to determine if LPNs can administer IV medications? 1Employer policy and procedures manuals 2Nursing faculty from a local nursing program 3The nurse practice act of the state in which the practice takes place 4American Nurses Association (ANA) professional standards - Correct answer 3 The nurse is assisting with the delivery of a newborn infant. What is the priority nursing intervention for a normal newborn immediately after delivery? 1Dry off infant with a warm blanket or towel 2Apply identification bracelets 3Assign the one-minute APGAR score 4Obtain vital signs - Correct answer 1Dry off infant with a warm blanket or towel - The registered nurse is teaching a childbirth education class about postpartum depression. Which statement, made by a class member, indicates that more teaching is needed? 1"I will make an effort to talk with someone about my feelings if I start to feel overwhelmed." 2"It's common for women with postpartum depression to have delusions about the infant." 3"Women with postpartum depression have feelings of guilt and worthlessness." 4"I may experience postpartum depression up to a year after delivery." - Correct answer 2 The nurse is reinforcing information about the side effects of fluoxetine to a client. Which group of findings should be included? 1Diarrhea, dry mouth, weight loss, reduced libido 2Tachycardia, blurred vision, hypotension, anorexia 3Orthostatic hypotension, vertigo, reactions to tyrosine, nausea 4Photosensitivity, seizures, edema, hyperglycemia - Correct answer 1Diarrhea, dry mouth, weight loss, reduced libido A client has a diagnosis of heart failure. Which intervention is most important for the nurse to implement prior to the administration of digoxin? 1Use the pulse reading from the electronic blood pressure device 2Take a radial pulse, counting for a full 60 seconds 3Check for a pulse deficit at least twice with another nurse 4Assess the apical pulse, counting for a full 60 seconds - Correct answer 4Assess the apical pulse, counting for a full 60 seconds - A client diagnosed with bipolar disorder refuses to take the prescribed medication. Which is the most therapeutic response by a nurse to the client's refusal of the medication? 1"You need to take your medicine. This is how you get better." 2"What is it about the medicine that you don't like?" 3"I can see that you are uncomfortable right now; let's talk about it tomorrow." 4"If you refuse your medicine, tell me how you think you will get better?" - Correct answer 2 A parent expresses frustration and anger about the toddler constantly saying "no" and refusing to follow directions. The nurse should help the parent understand that this behavior meets which developmental need? 1Self-esteem 2Initiative 3Independence 4Trust - Correct answer 3 The LPN is assisting the RN to provide care for a client diagnosed with a traumatic brain injury. Using the Glasgow Coma Scale, when the client does not obey verbal commands to move, which technique will the RN use to evaluate motor function? 1Squeeze the trapezius muscle firmly 2Lift the client's arm and observe for pronation and drift 3Apply fingertip pressure for 10 seconds 4Rub the sternum with the knuckles - Correct answer 1Squeeze the trapezius muscle firmly - A newborn has hyperbilirubinemia and is being treated with a biliblanket. Which intervention is indicated during this therapy? 1Discontinue breastfeeding during treatment 2Rotate the neonate to treat all of his/her skin 3Restrict holding the newborn during treatment 4Provide more frequent feedings - Correct answer 4Provide more frequent feedings- A client with paranoid delusions stares at the nurse over a period of several days. The client suddenly walks up to the nurse and shouts, "You think you're so perfect, pure and good." How should the nurse respond? 1"You seem to be in a bad mood." 2"Perfect? I don't quite understand." 3"You sound angry right now." 4"That explains why you've been staring at me." - Correct answer 3 The client with coronary artery disease has a prescription for nitroglycerin transdermal patches. What is the best reason the client should not wear a patch for more than 12 to 14 hours each day? 1It can cause severe headaches 2It may no longer work as well 3It will cause profound hypotensive effects 4it will irritate the skin - Correct answer 2 A hospitalized infant is receiving gentamicin. Which nursing intervention should receive priority in the plan of care? 1Compare daily infant weights 2Monitor the infant's urine output 3Ensure appropriate fluid intake 4Maintain accurate intake and output - Correct answer 2 A newborn is diagnosed with hypothyroidism. In discussing the condition and treatment with the family, the nurse should emphasize which point? 1They can expect the child will be mentally retarded 2Administration of a thyroid hormone will prevent problems 3This rare condition is hereditary 4Physical growth and development will be delayed - Correct answer 2 A client with heart failure is newly referred to a home health care agency. The nurse determines that the client has not been following the prescribed diet. It would be most appropriate for the nurse to take which action at this time? 1Notify the health care provider of the client's failure to follow the prescribed diet 2Make a referral to Meal-on-Wheels for delivery of one meal three times a week 3Discuss the diet with the client to learn the reasons for not following the diet - 4Recommend a release from home health care related to noncompliance - Correct answer 3 A client has chronic renal failure and is being treated at home. During weekly home visits, which factor is the most accurate indicator of fluid balance? 1Trends in daily weights - 2Skin turgor over at least two areas of the body 3Changes in mucous membrane moistness 4Difference between intake and output - Correct answer 1Trends in daily weights - The client is receiving a thrombolytic agent to open a clot-occluded coronary artery following a myocardial infarction. Which finding would be the greatest concern and should be immediately reported to the registered nurse? 1Hematemesis - 2Pink-tinged saliva 3Serosanguinous drainage from the IV site 4Slight rust-colored urine - Correct answer 1Hematemesis - The nurse is caring for a postoperative client following a closed reduction of distal tibia and mid- femur fractures. The client has a long leg plaster cast. Thirty-six hours after surgery, the client suddenly becomes confused, short of breath and spikes a temperature of 103 F (39.4 C). What should be the first action by the nurse? 1Check the distal circulation of the casted extremity 2Obtain the pulse oximetry reading 3Measure the client's blood pressure in the supine and Fowler's positions 4Check the orientation to time, place and person - Correct answer 2 The client has an order for intermittent gastrostomy tube (G-tube) feedings. What is the priority action by the nurse to accurately assess correct placement of the G-tube? 1Listen for active bowel sounds in all four quadrants 2Measure the pH of stomach content aspirate 3Auscultate the abdomen while instilling 10 mL of air into the G-tube 4Measure the length of tubing from the insertion site each shift - Correct answer 1Listen for active bowel sounds in all four quadrants 2Measure the pH of stomach content aspirate - 3Auscultate the abdomen while instilling 10 mL of air int1Listen for active bowel sounds in all four quadrants 2Measure the pH of stomach content aspirate - 3Auscultate the abdomen while instilling 10 mL of air into the G-tube 4Measure the length of tubing from the insertion site each shift the G-tube 4Measure the length of tubing from the insertion site each shift The client is diagnosed with infective endocarditis of the tricuspid valve. Which finding suggests a complication of this condition? 1Pronounced wheezes 2Pain on deep inspiration 3Sudden back pain 4Sudden dyspnea - Correct answer 4 A client is scheduled for a percutaneous Tran’s luminal coronary angioplasty (PTCA). What should the nurse understand about the purpose of this procedure? 1The surgical repair of a diseased coronary artery 2An noninvasive radiographic examination of the heart 3A process to compress arterial plaque to improve blood flow 4The placement of an automatic internal cardiac defibrillator - Correct answer 3 A 2 day-old infant born with spine bifida and meningomyocele is recovering after an initial surgery. As the nurse accompanies the grandparents for their first visit since the child's birth, which of these responses might the nurse expect from the grandparents? 1Anger 2Disbelief 3Depression 4Frustration - Correct answer 2 The ICU nurse works in a rural hospital that has a remote electronic ICU monitoring system (edict.) What is one of the best reasons for having access to an edict? 1An ICU nurse and intensives remotely monitor ICU clients around the clock 2An ICU nurse is on-call to answer questions when needed 3Clients can ask the intensives for a second opinion 4Less staff is needed on site when a remote edict is available - Correct answer 1 A child has severe burns to the lower extremities. A diet high in protein and carbohydrates is recommended. The nurse should care for this client with the knowledge that the most important reason for such a diet is to achieve which result? 1Provide a well-balanced nutritional intake 2Promote healing and strengthen the immune system 3Spare protein catabolism to meet metabolic and healing needs 4 stimulate increased peristalsis and nutrient absorption - Correct answer 3 A nurse is reinforcing information about the administration of an albuterol inhaler to an adult diagnosed with asthma. What should be the priority comment made by the nurse? 1"Use this medication at bedtime to promote rest." 2"Notify the health care provider if your canister lasts only two weeks." 3"Inhale this medication after other asthma sprays." 4"Discontinue the inhaler if you are dizzy." - Correct answer 2 An 80 year-old client is hospitalized for a chronic condition. The client informs family members that a living will has been prepared and the client wants no life-prolonging measures performed. The client's condition deteriorates and the client becomes unresponsive. Which of the following nursing actions is most appropriate? 1Notify the attending physician 2Consult the charge nurse and prepare to transfer the client to an intensive care unit 3Call the rapid response team 4Contact the family member indicated in the admission forms - Correct answer 1 The nurse is giving a morning bath to a client who has a colostomy. While giving the bath, the nurse should reinforce that the collection pouch should be emptied at what time? 1Prior to going to sleep at night 2After each fecal elimination 3At the same time each day 4When it is one-third to one-half full - Correct answer 4 A client is scheduled to have blood drawn for serum cholesterol and triglycerides tomorrow morning. What information should the nurse reinforce to the client about the test? 1"Be sure to eat a fat-free diet until the test, and drink lots of water." 2"Stay at the laboratory so that two blood samples can be drawn an hour apart." 3"Do not eat or drink anything but water for 12 hours before the blood test." 4"Have the blood drawn within two hours of eating breakfast." - Correct answer 3 The nurse is caring for a hospitalized adolescent. The nurse recognizes that which of these concerns will be the greatest for a hospitalized adolescent? 1Restricted physical activity 2Separation from family 3Altered body image 4Unrelieved pain - Correct answer 3 In checking a postpartum client, the nurse palpates a firm fundus. However, the nurse also observes a constant trickle of bright red blood from the vaginal opening. What should the nurse suspect? 1Retained placenta 2Clotting disorder 3Vaginal lacerations 4Uterine agony - Correct answer 3 A client diagnosed with gout is admitted with severe pain, swelling and redness in the proximal toe joint of the right foot. The nurse should anticipate that the plan of care would include which focus? 1High-protein diet 2Fluid intake of at least 3000 mL/day 3Acetaminophen for inflammation 4Hot compresses to affected joints - Correct answer 2 A 6 year-old child is hospitalized with findings of moderate edema, gross hematuria and mild hypertension associated with the diagnosis of acute glomerulonephritis (AGN). Which nursing intervention would be appropriate for this client? 1Weigh the child twice per shift 2Relieve boredom through physical activity 3Institute seizure precautions 4Encourage the child to eat protein-rich foods - Correct answer 3 A mother asks about expected motor skills for her 3 year-old child. Which activity should the nurse discuss as normal at this age? 1Riding a tricycle 2Tying shoelaces 3Jumping rope 4Playing hopscotch - Correct answer 1 The nurse is assessing a client who has been treated long-term with glucocorticoid therapy. Which finding might the nurse expect? 1Jaundice 2Peripheral edema 3Buffalo hump 4Increased muscle mass - Correct answer 3 A client diagnosed with autism begins to eat with both hands. The nurse can best handle the behavior by using which approach? 1Commenting "I believe you know better than to eat with your hands." 2Removing the food and stating "You can't have any more food until you use the spoon." 3Jokingly stating "Well, I guess fingers sometimes work better than spoons." 4Placing the spoon in the client's hand and stating "Use the spoon to eat your food." - Correct answer 4 The client is diagnosed with heart failure and oral digoxin is prescribed. What is the priority nursing assessment for this medication? 1Monitor serum electrolytes and creatinine 2Measure apical pulse prior to administration 3Maintain accurate intake and output ratios 4Monitor blood pressure every 4 hours - Correct answer 2 A 16 month-old child has just been admitted to the hospital. As the nurse assigned to this child enters the hospital room for the first time, the toddler runs to the mother, clings to her and begins to cry. What should be the next action of the nurse? 1Arrange to change client-care assignments 2Discuss with the parent the appropriate use of "time-out" 3Explain to the mother that the child needs extra attention 4Explain to the parent that this behavior is expected - Correct answer 4 The mother of a hospitalized 2 year-old child asks a nurse's advice about the child's screaming every time the mother gets ready to leave the hospital room. The best advice by the nurse would include which approach? 1Explain that this behavior will stop within a few days 2Suggest that the mother "sneak out" of the child's room when the child is asleep 3Request for the mother to remain with the child at all times 4Help the mother understand that this is a normal response to hospitalization - Correct answer 4 A client has a percutaneous endoscopic gastrostomy (PEG) tube that is used to administer feedings and medications. Which nursing action is best to ensure patency of the tube? 1Encouraging the client to cough to relieve abdominal bloating prior to or following a feeding 2Adequately flushing the tube with water before and after use 3Completely crushing all medications prior to administration 4Squeezing the tube to dislodge obstructions - Correct answer 2 A nurse is observing an 8 month-old client. Which behavior would the nurse anticipate the infant to be able to display? 1Pull up to stand 2Use a spoon 4Take a proton pump inhibitor either before or after eating - Correct answer 3 A practical nurse (PN) is observing an 8 month-old infant in the clinic waiting room. Which activity should be reported to the registered nurse (RN)? 1Lifts head from the prone position 2Rolls from abdomen to back 3Falls forward when sitting 4Responds to parents' voices - Correct answer 3 A nurse is monitoring the client's initial postoperative condition after a total thyroidectomy. Which findings should the nurse expect as complications and report immediately to the registered nurse (RN)? 1Paresthesia and muscle cramping 2Mild dysphagia and hoarseness 3Headache and nausea 4Irritability and insomnia - Correct answer 1 An 18 year-old client is admitted to intensive care from the emergency department after a diving accident. The injury to the spinal cord is suspected to be at the level of the second cervical vertebrae (C-2). When collecting data, which issue should be the priority focus? 1Muscle weakness 2Respiratory function 3Bladder control 4Peripheral sensation - Correct answer 2 There's a new order to apply one-inch of nitroglycerin paste to the client's chest every 12 hours, but the medication is not in the automatic medication dispensing system's drawer for this client. What should the nurse do next? 1Use another client's nitroglycerin paste until pharmacy sends a tube for this client 2Substitute an equivalent amount of nitroglycerin sublingual spray from the crash cart 3Call the pharmacy to send up a tube of nitroglycerin paste 4Call the prescriber and ask to substitute a different formulation of nitroglycerin - Correct answer 3 A nurse is caring for a child being discharged after a tonsillectomy. Which instruction is appropriate for the nurse to reinforce with the parents? 1Report a persistent cough to the health care provider 2The child can return to school in four days 3Administer chewable medication for pain 4The child may gargle as necessary for discomfort - Correct answer 1 An 80 year-old client is scheduled for a cardio version. The nurse is reviewing the client's medication administration records for the previous 24 hours. Which medication would prompt the nurse to notify the health care provider? 1Diltiazem (Cardizem) 2Digoxin (Lanolin) 3Nitroglycerine ointment 4Metoprolol tartrate (Toprol XL) - Correct answer 2 A nurse has reinforced teaching for a client who is being discharged after an arterial revascularization of the right lower extremity. Which statement made by the client is incorrect and requires further discussion with the nurse? 1"Smoking will decrease the circulation to my leg" 2"Coughing and deep breathing are important for a few weeks." 3"I will put my right leg through a full range of motion." 4"I might feel a throbbing pain in my right leg." - Correct answer 3 The nurse is assisting in the application of a plaster cast for a client with a broken arm. Which action is a priority? 1The cast material should be dipped several times into warm water 2The cast should be uncovered until it dries 3The casted extremity should be placed on a supporting surface 4The wet cast should be handled with the palms of hands for 48 to 72 hours - Correct answer 4 The client undergoes a gastrostomy. Several hours after surgery, the nasogastric (NG) tube stops draining. What action does the LPN anticipate the RN will take first? 1Reposition the tube 2Increase the amount of suction 3Gently irrigate the tube with sterile normal saline 4Notify the surgeon - Correct answer 3 A 12 year-old child, admitted with a broken arm, is waiting for a scheduled surgery. The nurse finds the child crying and unwilling to talk. What would be the most appropriate initial response by the nurse? 1Reassure the child that the surgery will go fine with no problems 2Provide privacy with encouragement to work through feelings 3Distract the child with a choice of activities to do while waiting for surgery 4Make arrangements for friends to visit as soon as possible - Correct answer 2 A nurse is caring for a client with a sigmoid colostomy. The client requests assistance in removing the flatus from a one-piece drainable stormy pouch. Which intervention should the nurse use? 1Pierce the plastic at the top of the stormy pouch with a pin to vent the flatus 2Pull the adhesive seal around the stormy pouch to allow the flatus to escape 3Open the bottom of the pouch to allow the flatus to be expelled 4Assist the client to ambulate to reduce the flatus in the pouch - Correct answer 3 A client returns from the operating room after a right orchiectomy. What is the priority nursing intervention during the immediate postoperative period? 1Manage postoperative pain 2Maintain fluid and electrolyte balance 3Control bladder spasms with PRN medication 4Ambulate the client within a few hours after surgery - Correct answer 1 The nurse enters the room of a postpartum mother and observes the baby lying at the edge of the bed while the mother sits in a chair. The mother states, "This is not my baby, and I do not want it." How should the nurse respond? 1"What a beautiful baby! The baby's eyes are just like yours." 2"This is a common occurrence after birth. Let's talk about how to accept the baby." 3"You seem upset, tell me about how you are feeling"? 4"Many women have postpartum blues and need some time to love the baby." - Correct answer 3 The client undergoes a laparoscopic removal of the appendix. Which postoperative instructions will the nurse reinforce? (Select all that apply.) - Correct answer may cause shoulder discomfort postoperatively. Clients should keep the dressings clean and dry for 48 hours before they can shower, but no tub baths for a few weeks. If "skin glue" is used over the incision(s), the client should not try to scrub it off because it will wear off on its own. Clients may resume normal activities as soon as they are able but no heavy lifting or aerobic exercise for about 2 weeks. If they do not have a BM after 2-3 days, clients can take 2 tablespoons of MOM several times a day until they have a BM. Diet can be advanced as tolerated but it's best to stick to non-greasy, non-spicy foods for a few days. The nurse is caring for a postoperative client. What is the priority nursing intervention the nurse will reinforce for preventing atelectasis? 1Turn, cough and breathe deeply 2Ambulate client within 12 hours 3Maintain adequate hydration 4Splint incision when moving or coughing - Correct answer 1 A nurse is working with parents to plan home care for a toddler with a heart problem. What should be the priority nursing intervention on the plan of care? 1Assist the parents to plan quiet play activities with the toddler at home 2Stress to the parents that they will need relief care givers 3Instruct the parents for them and the toddler to avoid contact with persons with infection 4Encourage the parents to enroll in child cardiopulmonary resuscitation (CPR) class - Correct answer 4 A client becomes acutely short of breath with a SpO2 (oxygen saturation) of 82%. Which oxygen delivery system should the nurse apply that would provide the highest concentrations of oxygen to the client? 1Simple face mask 2Partial rebreathe mask 3Venturi mask 4Non-rebreather mask - Correct answer 4 A nurse gathers data related to delayed gross motor development in a 3 year-old client. Which observation by the nurse should confirm this finding? 1Cannot ride a bicycle 2Cannot catch a ball 3Cannot skip on alternate feet 4Cannot stand on one foot - Correct answer 4 A client is admitted to the hospital with a history of confusion. The client has difficulty remembering recent events and becomes lost when outside of the home. Which statement would provide the best reality orientation for this client? 1"Hello. My name is Elaine Jones and I am your nurse for today." 2"Good morning. You're in the hospital. I am your nurse Elaine Jones." 3"How are you today? Remember, you're in the hospital. I will be your nurse all day. My name is Elaine Jones." 4"Good morning. I am Elaine Jones, your nurse. Do you remember where you are?" - Correct answer 2 A client is diagnosed with a Salmonella infection. What is a primary nursing intervention to be taken to minimize the transmission of disease from this client? 1Double glove when in contact with feces or emesis 2Wash hands thoroughly before and after any client contact 3Wear gloves when disposing of contaminated linens 4Use gloves when in contact with body secretions - Correct answer 2 A 6 month-old infant is being treated for developmental hip dysplasia and has been placed in a hip Spica plaster cast. Which discharge information is important for the nurse to reinforce with the parents? 1Turn the baby every two hours using the abduction stabilizer bar 2Check frequently for swelling in the baby's feet 3Gently rub the skin with a cotton swab to relieve itching 4Place favorite books and push-pull toys in the crib - Correct answer 2 A nurse is talking to parents about the side effects of routine immunizations. Which finding should the nurse reinforce about calling the health care provider if it occurs within 24 to 48 hours after a routine immunization? 1Localized tenderness at the injection site 2Tympanic temperature of 104 F (40 C) 3Some irritability and fussiness 4Swelling at the injection site - Correct answer 2 A 28-year-old is transferred to the emergency department (ED) via ambulance with a traumatic head injury. The client is awake and reports having a headache and some amnesia. What are the priority nursing interventions for this client? (Select all that apply.) - Correct answer Correct Response Assess vital signs and neurological function Assess the airway Prepare for CT imaging of the head Assess the wound for presence of drainage or bruising on the head A client exhibits many delusional thoughts. As the nurse assists the client to prepare for breakfast, the client comments, "Don't waste good food on me. I'm dying from this disease I have." Which response by the nurse would be the best? 1"None of the laboratory reports show that you have any physical disease." 2"Try to eat a little bit. Breakfast is the most important meal of the day." 3"I know you believe that you have an incurable disease." 4"What has your primary health care provider told you?" - Correct answer 3 The family member tells an admitting nurse that the client values the practice of Chinese medicine. The nurse must understand that for this family and client a priority goal should take which focus? 1Achieve harmony 2Respect life in old age 3Maintain energy balance 4Restore yin and yang - Correct answer 4 4"Testicular cancer has a very high cure rate with early diagnosis and treatment." - Correct answer 4 A nurse is caring for a child who has been recently diagnosed with cystic fibrosis. Which finding should the nurse anticipate? 1Dry, nonproductive cough 2Poor appetite 3Frequent urinary infections 4Ribbon-like stools - Correct answer 1 The nurse is caring for a client who is diagnosed with chronic renal failure with hemodialysis three times per week. The client becomes confused and irritable six hours before the next treatment. Which of these findings might explain the reason for the client's behavior? 1Low potassium level 2Elevated blood urea nitrogen (BUN) 3Low calcium level 4Metabolic alkalosis - Correct answer 2 The client is instructed to collect stool specimens at home using the guaiac test. In addition to explaining how to collect the specimens, the nurse instructs the client to avoid certain substances prior to obtaining the stool specimens. Which of the following substances should the client avoid? (Select all that apply.) - Correct answer a false positive test and should be avoided for at least 3 days before the fecal occult blood test; Fruits and vegetables with high peroxidase activity, such as red radishes, broccoli, and cauliflower should also be avoided several days prior to obtaining specimens. Clients should also limit their intake of vitamin C because too much can lead to a false negative result. A client with a fracture of the radius had a plaster cast applied two days ago. The client calls the clinic to report constant pain and swelling of the fingers since the cast was applied. What should be the next action of a nurse? 1Suggest to elevate the arm higher than heart level 2Ask if numbness is present in the fingers and if the client can move the fingers 3Have the client make an appointment with the surgeon for the next day 4Approve the application of a cool cloth to the fingers of the affected arm - Correct answer 2 The client is seen in the emergency one day after falling in his bathroom at home. The client reports having "a few drinks" prior to the fall. Which finding requires the nurse's immediate attention? 1Bruise behind one ear 2Blurred vision 3Nausea and vomiting 4Headache - Correct answer 1 Diagnosed with heart failure, the client had an implantable cardioverter-defibrillator (ICD) implanted several years ago. The client now has end-stage heart failure and is receiving home hospice care. Which end-of-life care option could have the greatest impact on client comfort? 1Encouraging the client to sit upright in bed 2Confirming advanced directives and plans for resuscitation 3Deactivating the implantable cardioverter-defibrillator (ICD) 4Assisting the client to eat several small meals - Correct answer 3 The client is prescribed alendronate (Fosamax). What information about medication administration should the nurse be sure to reinforce? 1Take on an empty stomach 2Take with milk, two hours after meals 3Take with calcium 4Take after meals - Correct answer 1 A couple experienced a miscarriage at seven months of pregnancy. The nurse makes a home visit one week after discharge from the hospital. What intervention should the nurse emphasize to the couple during the home visit? 1Plan another pregnancy as soon as possible 2Seek causes of the death for prevention purposes 3Focus on the other healthy children at home 4Discuss feelings with support persons and each other - Correct answer 4 A nurse is reinforcing information to a mother who is breast-feeding a newborn infant diagnosed with oral candidiasis. Which statement by the mother would be incorrect and indicate a need for reinforcement of information? 1"The therapy can be discontinued when the spots disappear." 2"I will boil the nipples and pacifiers for 20 minutes." 3"Expressed breast milk should be used immediately or frozen." 4"Nystatin should be given four times a day after my baby eats." - Correct answer 1 The nurse is to administer meperidine 100 mg, atropine 0.4 mg, and promethazine 50 mg IM to a client preoperatively. Which action should the nurse take initially? 1Place the bed in the low position 2Instruct the client to remain in bed 3Place the call bell within reach 4Have the client empty the bladder - Correct answer 4 The parents of a school-age child are providing information to the nurse about their child. Which of these health issues should the nurse recognize as a finding that could suggest type 1 diabetes? 1Being a picky eater 2Weight gain 3Bedwetting 4Oily and acne-prone skin - Correct answer 3 An adolescent client arrives at a clinic three weeks after the birth of her first baby. She tells the nurse she is very worried about not returning to her pre-pregnancy weight. Which approach should the nurse take first? 1Review the client's pattern of weight gain over the past year 2Encourage her to talk about her self-image 3Give her several pamphlets on postpartum nutrition 4Ask the mother to record her diet for the next few weeks - Correct answer 2 A nurse is caring for a client admitted with the diagnosis of suspected Legionnaire's disease. Which finding would require the nurse's immediate attention? 1Dry mouth with frequent requests for water 1Clinical specialty certification by an accredited organization 2Complete and accurate documentation of assessments and interventions 3Sworn statement that health care provider orders were followed 4Above-average performance reviews prepared by nurse manager - Correct answer 2 The nurse is assigned to care for several clients on the day shift. Which client should the nurse see first after receiving shift report? 1The client with asthma who is scheduled for a chest X-ray prior to discharge 2The client with peptic ulcer disease who has been vomiting most of the night 3The client with chronic kidney disease who completed peritoneal dialysis two hours ago 4The client with pancreatitis who reports pain at a level of eight out of 10 - Correct answer 2 The nurse hears a health care provider (HCP) loudly criticizing one of the unlicensed assistive persons (UAP) within the earshot of others. The UAP does not react or respond to the HCP's complaints. What is the best action by the nurse? 1Notify the chief of the medical staff about the HCP's breach of professional conduct. 2Encourage the UAP to directly confront the HCP about the unprofessional behavior. 3Complete an incident report describing the HCP's unprofessional behavior. 4Walk up to the HCP and quietly state, "This unacceptable behavior has to stop." - Correct answer 2 Information about case management and the role of the case management nurse is presented during an orientation session for new nurses. Which statement correctly describes an important fact about case management? 1Case management strategies focus mainly on the client's needs after discharge. 2Case management is a collaborative process designed to meet complex client needs. 3Physicians are responsible and accountable for client outcomes. 4The interdisciplinary team makes all the decisions for the client and family. - Correct answer 2 During the management of a client's pain, the nurse should adhere to the code of ethics for nurses. Which of these actions should the nurse consider first when treating the client's pain? 1Cultural sensitivity is fundamental to client-centered pain management. 2Clients have the right to have their pain managed promptly. 3Nurses should not judge a client's pain based on the nurse's values. 4The clients self-report of pain is the most important consideration. - Correct answer 4 A client with a musculoskeletal disorder has been newly fitted with a lower limb orthotic. Which activity can the nurse delegate to the certified nursing assistant (CNA)? 1Provide instruction to the client for ambulation with the orthotic. 2Monitor the client's response to moving with the orthotic. 3Check the client's skin for any redness or irritation from the orthotic. 4Assist with transferring the client from the bed to the chair. - Correct answer 4 Upon completing a review of a 27-year-old client's admission documents, the nurse identifies that the client does not have an advance directives. What action should the nurse take? 1Lecture the client on the importance of having advance directives. 2Inform the charge nurse to offer information about advance directives. 3Advance directives are not appropriate for this client due to the client's age. 4Refer this issue to the client's health care provider. - Correct answer 2 The home health nurse is visiting a client diagnosed with type 1 diabetes and osteoarthritis. The client has difficulty holding and using the prescribed insulin pen. The nurse should refer the client to which community resource person? 1Physical therapist 2Pharmacist 3Physical therapist 4Occupational therapist - Correct answer 4 Holding and using an insulin pen requires fine motor skills and good vision. A client with osteoarthritis (OA) might experience limited movement and pain in the joints of the fingers and hand. An occupational therapist can help a client improve the fine motor skills needed to prepare an insulin injection. An occupational therapist works with clients to perform tasks that are needed for smaller movements to maintain activities of daily living or for work. A client diagnosed with schizophrenia insists that the nurse explain the use and side effects of the medications prescribed for the client. What should the nurse understand before responding to the client? 1The psychiatrist will need to grant permission to discuss the client's medications. 2All clients have a right to be informed about their prescribed medications 3A decision to reinforce or not reinforce information about medications should be made by the nurse alone. 4It is too dangerous for clients who are diagnosed with schizophrenia to know about their medications. - Correct answer 2 The nurse asks another staff nurse to sign for wasting a partial-dose opioid injection, although the wasting was not witnessed by anyone. This type of request seems to be a pattern of behavior for this nurse. What is the most appropriate action for the second staff nurse to take? 1Report this request immediately to the nurse manager. 2Review the client's medication administration record (MAR) for past wastes. 3Ask the nurse's client if they witnessed the waste of the partial dose. 4Confront the nurse about suspected narcotics diversion. - Correct answer 1 A client diagnosed with bipolar disorder has been referred to social services for possible placement in a community halfway house after discharge. The social worker telephones the nurse and asks for information about the client's mental status and adjustment. What should the nurse do next to respond to this request? 1Go ahead and provide the information, since the client is ready for discharge. 2Inform the caller that this kind of information is never given over the telephone. 3Refer the social worker to the health care provider to obtain the requested information. 4Verify that the client's medical record includes the client's written consent to release information. - Correct answer 4 During a discussion about a living will, the client's son states, "I do not understand the need for a living will." What is the best response by the nurse? 1"Health care decisions can be made based on the client's wishes." 2"Specific instructions are listed for specific diseases." 3"A designated family member can make all decisions." 4"Do not resuscitate (DNR) orders are automatic under these conditions." - Correct answer 1 The client requests not to be interrupted before 10 am because it interferes with the client's time to meditate. What action shall the nurse take first? 1Document the client's request in the medical record. 2Meet with the client to formulate a mutually agreeable schedule. 4Wash hands thoroughly with soap and warm water after contact with the child. - Correct answer 4 The hepatitis A virus spreads through contaminated food or water, as well as unsanitary conditions in childcare facilities or schools. The infection resolves spontaneously and symptom relief is usually the only treatment. A client has been placed in physical restraints due to aggressive behavior. Which of the following demonstrates that the nurse has appropriately implemented the restraints? (Select all that apply.) - Correct answer to avoid injury, restraints should never be fastened to a moving part of a bed or stretcher. A physical restraint order is never "as needed." An order must be written by a provider for each restraint episode. Documentation must be done every 15 minutes on the restraint flow sheet, which is part of the client's permanent medical record. It is a legal requirement to notify the client's advocate or a relative if requested by the client. The nurse is reinforcing education to a group of parents on how to treat accidental poisoning of children in the home. What information should the nurse include? 1Empty the child's mouth of any poisonous substance still present. 2Give the child a glass of milk to drink to neutralize the poisonous substance. 3Induce vomiting if the child is suspected of swallowing something poisonous. 4Start treatment before calling the Poison Control Center - Correct answer 1 The nurse is stuck in the hand by an exposed needle that was accidentally left in the client's bed. What action should the nurse take first? 1Contact employee or occupational health services. 2Look up the policy and procedure on needle stick injury. 3Immediately wash hands vigorously with soap and warm water. 4Notify the nursing supervisor and complete an incident report. - Correct answer 3 The nurse is reviewing the documentation of a client's care in their electronic health record and realizes that one of the entries was completed on the wrong client. Which of the following actions are appropriate for the nurse to take? - Correct answer Mark the entry as "mistaken entry-wrong patient." Enter the time the error was discovered. The nurse is preparing a client for a colonoscopy and notes that the consent form has not been signed. Which of the following statements by the nurse are appropriate to make to the client? - Correct answer "Please tell me your full name and date of birth." "Do you have any questions about the colonoscopy?" "Describe what the health care provider told you about a colonoscopy." An outpatient client is scheduled to receive an oral solution of radioactive iodine. In order to reduce radiation exposure to others, which information should the nurse reinforce? 1No solid food may be eaten for six hours after ingestion. 2Urine and saliva will be radioactive for 24 hours after ingestion. 3Wash laundry separately and rinse twice in hot water. 4Wait for 48 hours to have grandchildren visit at home. - Correct answer 2 The client is diagnosed with active tuberculosis (TB) and the case has been reported to the local health department. The nurse understands that the most important reason for notifying the health department is: 1To ensure that treatment compliance will be monitored 2To trace and screen recent contacts the client had 3To maintain important disease outbreak statistics 4To track the incidence of tuberculosis cases - Correct answer 2 The parent of a toddler who is being treated for suspected poisoning asks, "Why is activated charcoal used?" What is the best response by the nurse? 1"When the poison is absorbed into the blood stream, the activated charcoal will neutralize it." 2"Activated charcoal binds with the poison to limit absorption in the digestive tract." 3"Activated charcoal causes vomiting, which will eliminate the poison from the body." 4"The activated charcoal will protect the kidneys from any long-lasting damage." - Correct answer 2 The parents of a toddler ask, "How long will our child have to sit in a car seat when riding in a car?" What would be the best response by the nurse? 1"Until the child is able to sit in a booster seat." 2"Until the child weighs 40 pounds." 3"Until the child outgrows the car seat." 4"Until the child is 50 inches tall." - Correct answer 3 The nurse has administered haloperidol 5 mg orally (PO) as needed (PRN) to a client with a diagnosis of schizophrenia. Which of the following behaviors justify use of this chemical restraint? - Correct answer the client is verbalizing a plan to harm another client. The client is expressing paranoid delusions. The client is experiencing command hallucinations. A community health clinic nurse is interviewing a client who is experiencing lightheadedness. The client reports a history of arthritis and is taking naproxen sodium for the pain. The client is pale, the blood pressure is 88/40, pulse is 114, respiratory rate is 22 and temperature is 98.2° F (36.7 C°). What additional information should the nurse solicit from the client? (Select all that apply.) - Correct answer frequency and amount used Color of bowel movements Bruising The nurse is discussing modifiable cardiac risk factors with a group of adult clients at a community center. Which topic should the nurse reinforce as the highest priority intervention? 1Increasing physical exercise 2Smoking cessation 3Stress management 4Weight reduction - Correct answer 2 The home health nurse is seeing a client diagnosed with type 2 diabetes. The client has a small foot ulcer that was debrided and requires daily dressing changes. Which intervention is most important for the nurse to implement to meet the goal of uncomplicated wound closure? 1Schedule regular visits to monitor wound healing. 2Involve the client in making decisions. 3Evaluate the client's understanding of appropriate foot care. 4Arrange for referral to a diabetic educator. - Correct answer 2 The client is in her first trimester of pregnancy. What major developmental task should the client accomplish during this stage of pregnancy? 1Viewing the fetus as a separate and unique being. A 20-year-old male client who has a profuse, purulent urethral discharge with painful urination is seen at a community health clinic. Which information will be most important for the nurse to obtain? 1Sexual orientation 2Recent sexual contacts 3Immunization history 4Contraceptive preference - Correct answer 2 The nurse recognizes that which finding indicates a child has attained the developmental stage of concrete operations, according to Piaget? 1The child makes the moral judgment that "stealing is wrong." 2The child explores the environment with the use of sight and movement. 3The child thinks in mental images or word pictures. 4The child reasons that homework is time-consuming but necessary. - Correct answer 1 After the death of a client, the family approaches the nurse and requests that a family member be allowed to perform a ritual bath on the deceased client prior to moving the body. What would be the most appropriate response by the nurse? 1"I will have to check on hospital regulations and policies." 2"These procedures have to be carried out by our staff." 3"Is there anything you need from me to perform the bath?" 4"A ritual bath will have to wait until after postmortem care." - Correct answer 3 A nurse is working to establish a therapeutic relationship with a client. Which action would support the nurse's goal? 1Establish trust and rapport with the client. 2Identify with what the client is feeling. 3Praise the client for appropriate behavior. 4Advise the client on problem-solving techniques. - Correct answer 1 The client diagnosed with paranoid-type schizophrenia is sitting alone, intently staring at and watching other clients and staff members. The client becomes hostile when approached with medication and claims that the medication controls the mind. What type of symptom(s) does the nurse recognize that this client is exhibiting? 1Antisocial behavior 2Negative symptoms 3Positive symptoms 4Inappropriate affect - Correct answer 3 Symptoms of schizophrenia are commonly described as positive or negative. Positive symptoms are behaviors and experiences present in a person with schizophrenia that would not be present in a person without the illness. These are sometimes described as features that are "added" by the illness. In contrast, negative symptoms are those that reflect a decrease in normal functions, or abilities that have been "taken away." Positive symptoms of schizophrenia include delusions, hallucinations, hyper vigilance and disorganized thinking. The nurse is providing care for a client who has been diagnosed with terminal cancer. The nurse notes that the client's wife is not visiting very often. When she does visit the client, she only stays for a brief time, stands in the corner and does not approach the client during interactions. Which of the grieving processes is the client's wife most likely experiencing? 1Disenfranchised grief 2Anticipatory grief 3Perceived loss 4Death anxiety - Correct answer 2 Anticipatory grief is the family member becomes distant and detached from the client and the client feels isolated and alone. Death anxiety is worry or fear related to dying that may be seen with a grieving child. Disenfranchised grief is when the individual cannot acknowledge the loss, perhaps because of an unrecognized loss, such as an abortion or a suicide. Perceived loss is a loss that cannot be verified by others such as a loss of self-esteem or a loss of control. The nurse is caring for a postpartum Latina client who keeps declining the hospital food because it is "cold." What action should the nurse take initially? 1Send the food to be reheated. 2Encourage the client to eat for strength. 3Ask the client what foods are acceptable. 4Consult with the dietitian as soon as possible. - Correct answer 3 An adolescent client is paralyzed from the waist down after being involved in a motor vehicle accident. Which client statement would indicate to the nurse that the client is using repression as an ego defense mechanism? 1"It's all the other driver's fault! They were driving too fast." 2"I don't remember anything about what happened to me." 3"My parents are heartbroken about my situation." 4"I know that I will walk again one day." - Correct answer 2 Repression is the unconscious and involuntary forgetting of painful events, ideas and conflicts. The nurse is working with a couple who is experiencing intense anxiety after their home was completely destroyed by a fire. The nurse should implement which initial intervention? 1Suggest finding an apartment with a sprinkler system. 2Explore the couple's feelings of grief and loss. 3Determine what community housing resources are available. 4Provide a brochure on relaxation and stress relief. - Correct answer 3 The couple has experienced a crisis, i.e., sudden loss event that has resulted in disequilibrium. The most important initial crisis intervention focuses on identifying resources and obtaining assistance for housing and other immediate needs. The nurse is evaluating a client who is being physically abused by the client's domestic partner. The client states, "I need a little time away." Which is the most likely response from the partner for which the nurse should prepare the client? 1Fear of rejection, resulting in increased rage toward the client 2Relief over a separation as a way to have some personal time 3Acceptance and understanding that the relationship is in trouble 4A new commitment to seek counseling to assist with problems - Correct answer 1 A client is admitted to the medical-surgical unit following a motor vehicle accident. Twelve hours after admission the client becomes diaphoretic, tremulous and irritable, and the client's pulse and blood pressure are elevated. The client states to the nurse, "I have to get out of here." What is the most likely cause for the client's symptoms and behavior? 1Dissatisfaction with hospital care 2Anxiety related to being hospitalized 33hock related to the injuries mumbling to themselves and speaking to the wall. Which priority goal/outcome should the nurse select for the client's plan of care? 1Client will express feelings appropriately through verbal interactions. 2Client will accurately interpret events and other's behaviors. 3Client will engage in meaningful and understandable verbal communication. 4Client will demonstrate improved social relationships. - Correct answer 3 The nurse is working in an inpatient psychiatric setting and understands that touching clients should be limited to a quick handshake for which reason? 1Touching a client, other than a handshake, can set off a violent episode. 2Refraining from touching signals the termination of the nurse-client relationship. 3A handshake allows the use of therapeutic touch while maintaining boundaries. 4A handshake will not be misinterpreted as an invitation to more sexual behavior. - Correct answer 3 The nurse in a behavioral health inpatient unit is observing a female client who has been diagnosed with obsessive-compulsive disorder (OCD). Which behavior should the nurse expect to see with this diagnosis? 1The client is seen washing her hands every 15 minutes. 2The client exhibits repetitive, involuntary movements. 3The client verbalizes suspicions about thefts on the unit. 4The client prefers to interact with female staff members. - Correct answer 1 OCD is characterized by repetitive, unwanted, intrusive thoughts (obsessions) and irrational, excessive urges to perform certain actions (compulsions). The nurse is caring for a client who has an alcohol use disorder (AUD). The client states that the client's dysfunctional family caused the addiction. Which response by the nurse would best help the client accept responsibility for their own behavior? 1"It was your choice to drink, so you need to take responsibility." 2"It is wrong for you to blame your problems on your family." 3"Yes, I can understand that families can be tough to deal with." 4"The lab report showed a high blood alcohol level when you were admitted." - Correct answer 4 A nurse is collecting data on a client believed to be in an abusive relationship. Which client statement is most indicative that this individual is experiencing intimate partner abuse? 1"I must have done something to deserve this." 2"No one else in my family has been treated like this." 3"I have only been in this relationship for two months." 4"I will keep praying that things will get better." - Correct answer 1 A home health nurse is caring for a client diagnosed with late-stage, Lowy body dementia (LBD). The nurse is meeting with the client's family to discuss options for care of the client. What is the initial question the nurse should ask to assist the family with their decision-making process? 1"What is your opinion of nursing homes or assisted living facilities?" 2"Is your parent currently taking over-the-counter (OTC) or prescription medications?" 3"Are you able to assist with the care of your parent in any manner?" 4"What type of assistance does your parent require?" - Correct answer 4 The nurse is caring for a client with paraplegia due to a spinal cord injury at the T-7 level. Which nursing intervention should be a priority for this client? 1Obtain a pressure-reducing mattress for the client's bed. 2Observe the client performing self-catheterization correctly. 3Consult with the discharge planner about equipment the client's needs at home. 4Encourage the client to increase intake of fluids and high-fiber foods. - Correct answer 1 The nurse is evaluating the plan of care for a client with osteoporosis. What type of activity should the nurse reinforce for this client? 1Enroll in a kickboxing class twice a week. 2Walk for 30 minutes, 3 to 5 times a week. 3Participate in swimming lessons three times a week. 4Go jogging 5 to 7 times a week. - Correct answer 2 Teach the client (or reinforce teaching) that walking for 30 minutes, 3 to 5 times a week, is the single most effective exercise for osteoporosis prevention. The nurse is caring for child diagnosed with celiac disease. Which of the following foods would be an appropriate snack choice for this child? 1A cup of cereal 2A slice of wheat bread 3A cup of yogurt 4An oatmeal cookie - Correct answer 3 A client is on NPO status and has a nasogastric (NG) tube in place, connected to low- intermittent suction, to help resolve a small bowel obstruction. Which nursing intervention should the nurse implement for this client? 1Allow the client to melt ice chips in their mouth. 2Provide oral care at least every 2 to 4 hours. 3Swab the client's mouth, using glycerin swabs. 4Provide the client mints to freshen their breath. - Correct answer 2 The nurse is caring for an adult client who suffered second degree burns over 25% of their body in a house fire. Which observation best indicates that fluid resuscitation has been effective? 1Elastic, contenting skin turgor 2Moist oral mucus membranes 3Urine output of 35 mL per hour 4No reports of thirst - Correct answer 3 The goal is to maintain an hourly urine output of 0.5 mL/hour (about 30 mL/hour) for the average adult. Which of the following actions by the nurse indicates a need for additional education on the prevention of health care-associated infections (HAIs)? 1The nurse uses their own stethoscope to assess the lung sounds of a client placed on contact precautions for Methicillin-resistant Staphylococcus aurous (MRSA) infection. 2The nurse calls the health care provider (HCP) to request the removal of the indwelling urinary catheter for a two days postoperative client. 3The nurse cleanses hands with soap and water for 60 seconds after caring for a client with Clostridium difficult (C. difficult) infection. 4The nurse wears a gown and gloves when providing perinea care to a client with Vancomycin- resistant Enterococci (VRE) infection. - Correct answer 1 The nurse is evaluating a client who has been diagnosed with heart failure (HF) to gauge their understanding of the required diet modifications. Which menu items selected by the client indicate to the nurse that the client understood the teaching? 1Cheeseburger and baked potato chips 2Grilled cheese sandwich with a glass of skim milk 3Leftover turkey on a sandwich and fresh pineapple 4Vegetable pizza and ice cream - Correct answer 3 Clients with HF should adhere to a low-sodium diet to prevent fluid volume excess. A sodium- restricted diet should consist of less than 2 grams of sodium per day. (A regular diet should include 4 to 6 grams of sodium per day.) The nurse receives an order to give a client iron by deep injection. What does the nurse understand about the reason for using this method of administration? 1Provides more even distribution of the drug 2Prevents the medication from tissue irritation 3Ensures that the entire dose of medication is given 4Enhances absorption of the medication - Correct answer 2 Deep injection, or Z-track, is a special method of giving medications via the intramuscular route. Use of this technique prevents irritating or staining medications from being tracked through tissue. A client has been taking alprazolam for three days. For which expected effect of the medication should the nurse evaluate the client? 1The client reports feeling less depressed. 2The client reports sleeping through the night. 3The client denies having auditory hallucinations. 4The client denies having suicide ideation. - Correct answer 2 Antianxiety medications or anxiolytics, such as alprazolam, a benzodiazepine, work quickly. They produce sedative effects and reduce anxiety through effects on the limbic system, a neuronal network associated with emotionality. They also promote sleep through effects on cortical areas and on the brain's sleep-wakefulness "clock." At 9 am, the nurse administers 10 units of insulin apart subcutaneously to a client with a blood sugar of 322 mg/LD. At approximately what time should the nurse expect the insulin to peak? 1At noon 2At 9:30 am 3At 10:00 am 4This insulin does not peak because it acts over 24 hours. - Correct answer 1 Insulin apart is an analog of human insulin with a rapid onset (10 to 20 minutes) and short duration (3 to 5 hours). Injections should be given 5 to 15 minutes before meals. A nurse notes an abrupt onset of confusion in an 85-year-old client. Which newly prescribed medication most likely caused this change in the client's mental status? 1Diphenhydramine 2Metoprolol 3Warfarin 4Pantoprazole - Correct answer 1 An older adult client is to receive intravenous (IV) gentamicin for urosepsis. Before administering the medication, for which finding should the nurse notify the health care provider (HCP)? 1The client has a history of retinopathy. 2The client has a history of chronic kidney disease. 3The client has a history of acid reflux disease. 4The client has a history of urinary retention. - Correct answer 2 They are excreted by glomerular filtration. Aminoglycosides are nephrotoxic and requires close monitoring of renal function. A client with chronic kidney disease should not receive this medication. The nurse in a long-term care facility is preparing to administer medications. Which physiological changes does the nurse know will affect medication pharmacokinetics in older adults? 1Due to an increase in glomerular filtration rates, medications are excreted more rapidly. 2Due to a decrease in gastric emptying, higher medication doses are prescribed. 3Due to a decrease in renal drug excretion, a greater risk for adverse medication effects exist. 4Due to an increase in metabolism, medications are prescribed more frequently. - Correct answer 3 The nurse prepares to administer a liquid medication to an infant. At the bedside, the parent states that the infant does not like to take medications. Which action should the nurse perform to ease the medication administration? 1Use an oral syringe to administer the medication, alternating with a pacifier. 2Mix the liquid medication with a full bottle of formula. 3Give half the dose now and the remaining amount in an hour. 4Ask the health care provider (HCP) to switch the medication to an injection. - Correct answer 1 A client is admitted with deep vein thrombosis (DVT). The health care provider (HCP) orders the immediate administration of an intravenous bolus of heparin sodium 200 units/kg. The client weighs 187 lbs. How many mL should the nurse draw up from the supplied 10 mL vial that contains 5,000 units per mL? Do not round. - Correct answer 3.4 The nurse has given discharge instructions to a client who suffers from sensory neuropathy due to diabetes. The client was prescribed gabapentin. Which of the following statements indicates that the client understands the nurse's instructions regarding the medication? 1"I can stop taking the medication at any time." 2"It is safe to take extra doses if my pain becomes worse." 3"The medication might cause me to have insomnia." 4"My doctor prescribed it for the pain in my legs." - Correct answer 4 The nurse is reviewing medication orders for a client who has requested something for pain. In the process, the nurse finds a new written order for a pain medication. The health care provider (HCP) wrote, "Give APAP every six hours as needed for pain." Which parts of the medication order should the nurse clarify before administering the medication? - Correct answer route Drug name Dosage The nurse administers a medication to the wrong client. Which action(s) should the nurse take when the medication error is identified? (Select all that apply.) - Correct answer Notify health care provider Complete an incident report Monitor the client for adverse effects The nurse is preparing to administer an antibiotic intramuscularly (IM) to a 2-year-old child. The total volume of the injection is 2 mill what is the best approach for the nurse to take when administering this medication? 1Call the provider and request a smaller dose. 2Split the medication into two separate injections. 3Substitute an oral form of the medication. 4Inject the medication in the deltoid muscle. - Correct answer 2 The nurse is reinforcing teaching for a client with chronic kidney disease about the prescribed aluminum hydroxide. Which is the best statement by the nurse about this medication? 1"It reduces potassium levels." 2"It increases urine output." 3"It controls stomach acid secretions." 4"It decreases phosphate levels." - Correct answer 4 Phosphates tend to accumulate in the client with chronic kidney disease due to decreased filtration capacity of the kidneys. Antacids that contain aluminum such as aluminum hydroxide (Asphodel) are commonly used to lower phosphate levels. A client has a new prescription for sertraline, a selective serotonin reuptake inhibitor (SSRI) antidepressant. After reviewing the client's medical record, which data is the nurse most concerned about? 1History of an eating disorder 2Current prescription for phenelzine 3History of premenstrual dysphonic disorder 4Current prescription for alprazolam - Correct answer 2 Phenelzine is a monoamine oxidase inhibitor (MAOI) antidepressant. Combining MAOIs with SSRIs and other serotonergic drugs poses a risk of serotonin syndrome. The client is discharged from the hospital with a new prescription for furosemide. During a follow-up visit one week later, the nurse notes the following findings. Which finding is most important to report to the health care provider? 1Increased urine production 2Occasional lightheadedness 3Muscle cramps 4Constipation - Correct answer 3 Furosemide is a loop (potassium-wasting) diuretic. It can cause dehydration and hypokalemia, which can result in muscle cramps. This is the most important finding. The nurse is reviewing prescribed medications with a client. Which information should the nurse reinforce about captopril? 1Avoid using salt substitutes. 2Avoid green leafy vegetables. 3Restrict fluids to 1000 mL/day. 4Take the medication with meals. - Correct answer 1 Captopril is an angiotensin converting enzyme (ACE) inhibitor. It reduces aldosterone secretion, thereby reducing sodium and water retention. Captopril is used to treat hypertension and heart failure. Because it can cause an accumulation of serum potassium (i.e., hyperkalemia), clients should avoid the use of salt substitutes, which often contain potassium instead of sodium chloride. A nurse administers cimetidine to a 75-year-old client diagnosed with a gastric ulcer. The nurse should monitor the client for which adverse reaction? 1Hearing loss 2Mental status change 3Constipation 4Increased liver enzymes - Correct answer 2 Cimetidine is a histamine H2-receptor antagonist used to treat gastric ulcers. It has been found to cause confusion in susceptible clients, such as the elderly and debilitated clients. Clients over age 50 or who are severely ill may become temporarily confused while taking H2 blockers, especially cimetidine. A client at risk for a stroke has been prescribed clopidogrel. Which information is most important for the nurse to reinforce with the client? 1"You must take the medication on an empty stomach." 2"If you miss a dose, take a double dose the next day." 3"You must have your lab tests checked weekly." 4"You must stop the medication a week before your surgery." - Correct answer 4 Clopidogrel is an oral antiplatelet drug with similar effects to aspirin. The drug is taken for secondary prevention of myocardial infarction, ischemic stroke and other vascular events. Clopidogrel prevents platelet aggregation. Like all other antiplatelet drugs, clopidogrel poses a risk of serious bleeding. Clopidogrel should be discontinued 5 to 7 days before elective surgery. The nurse is reinforcing the correct use of a metered-dose inhaler (MDI) for a client newly- diagnosed with asthma. The client asks, "How will I know the canister is empty?" What is the best response by the nurse? 1"Drop the canister in water to observe if it floats." 2"Contact your pharmacy to find out when to obtain a refill." 3"Count the number of doses as the inhaler is used." 4"Shake the canister and listen for any fluid movement." - Correct answer 3 A client is prescribed furosemide and digoxin for heart failure. The nurse should monitor the client for which potential adverse drug effect? 1Pulmonary hypertension 2Acute arterial occlusion 3Cardiac dysrhythmias 4Acute kidney injury - Correct answer 3 Digoxin is a cardiac glycoside, or positive inotrope that increases myocardial contractility. By increasing contractile force, digoxin can increase cardiac output in clients with heart failure (HF). Furosemide is a potassium-wasting (loop) diuretic, prescribed to prevent fluid overload in clients with HF. Clients who take furosemide are at risk for developing hypokalemia. Potassium ions compete with digoxin and a low potassium level can cause digoxin toxicity, leading to lethal cardiac dysrhythmias. Therefore, it is imperative that potassium levels be kept within normal range (3.5 to 5 me/L) while taking digoxin. The nurse in an ambulatory clinic is speaking with the parents of a 2-year-old child diagnosed with acute otitis media. Which information is most important for the nurse to include in the instructions to the parents? 1The child must complete the entire course of the prescribed antibiotic. 2The child may be given a decongestant to relieve pressure on the tympanic membrane. 3The child should return to the clinic to evaluate effectiveness of the treatment. 4The child may be given acetaminophen or ibuprofen drops for pain. - Correct answer 1 The therapeutic range for INR is 2 to 3, therefore a client with a 5.5 INR is at a high risk for bleeding and the nurse should notify the HCP immediately. A nurse is caring for a 2-year-old child who underwent a tonsillectomy at 8:00 am. At 11:00 am, the child has a temperature of 98.2 F (36.7 C). At 1:00 pm, the child's parent reports to the ⁰ ⁰ nurse that the child feels very warm to touch. What should the nurse do first? 1Reassure the parent that this is normal. 2Take the child's temperature. 3Offer the child cold oral fluids. 4Administer prescribed acetaminophen. - Correct answer 2 The nurse is caring for a 60-year-old female client scheduled for abdominal surgery. Which factor in the client's history indicates that the client is at an increased risk for deep vein thrombosis (DVT) in the postoperative period? 1Past hypersensitivity to heparin 2Family history of uterine cancer 3Estrogen replacement therapy for the past three years 4History of acute hepatitis A - Correct answer 3 The estrogen in hormone replacement therapy (and in birth control pills) can increase clotting factors in the blood, increasing the risk for development of a DVT. The nurse is caring for a client receiving mechanical ventilation. The nurse understands which are the possible causes for a high-pressure alarm? (Select all that apply.) - Correct answer Kinked tubing, secretions and/or bronchospasms cause obstruction to airflow from the ventilator, creating high pressure in the ventilator circuit and setting off the high-pressure alarm. The nurse is reviewing the medical record of a client on the medical surgical unit and notes a positive result of the stool for occult blood (OB) test. The nurse recognizes which risk factors for this result? (Select all that apply.) - Correct answer Drugs that can cause GI bleeding include NSAIDs such as ibuprofen and naproxen (Aleve). Corticosteroids can cause gastric irritation, including peptic ulcers that can also lead to GI bleeding. Factors that may cause a false positive result include bleeding gums following a dental procedure and the ingestion of red meats within three days before testing because red meats contain animal hemoglobin. A client is scheduled for a computerized tomography (CT) scan of the abdomen with contrast. What action should the nurse take before sending the client to the imaging department? 1Insert a temporary urinary catheter. 2Confirm that a signed consent is in the chart. 3Keep the client on bed rest. 4Hold all of the client's medications. - Correct answer 2 An 80-year-old client with type 2 diabetes mellitus is admitted to the emergency department with worsening confusion and decreased level of consciousness. Which of these findings is most important for the nurse to report to the health care provider? 1Blood glucose of 380 mg/LD. 2Arterial blood pH of 7.36 3Urine output greater than 100 mL/hour 4Serum osmolality of 355 mom/L - Correct answer 4 The nurse is monitoring a 45-year-old client who just underwent a cardio version for dysrhythmias. The client's respirations are 12 per minute. Which action should the nurse take next? 1Measure the client's oxygen saturation. 2Ask another nurse to verify the respiratory rate. 3Notify the health care provider (HCP). 4Continue to monitor the client. - Correct answer 4 Normal respirations range from 12 to 20 per minute; respirations of eight or less per minute would be a cause for concern. The nurse is preparing a client for an intravenous pyelogram (IVP) test. Which intervention should the nurse plan to implement? 1Limit client's fluid intake to 400 mL prior to the test. 2Inform client that no special preparation is necessary. 3Instruct client to maintain a regular diet until the test. 4Administer a laxative the evening before the test. - Correct answer 4 It is important for the large intestine to be clear of stool to allow full visualization of the kidney, bladder and ureters. The nurse is preparing to suction a client's tracheostomy. What action should the nurse take to prevent hypoxia during the procedure? 1Explain procedure to client. 2Monitor heart rate during suctioning. 3Use sterile technique. 4Provide preoxygenation to the client. - Correct answer 4 The nurse is caring for a comatose client. To prevent keratitis, moisturizing ointment should be prescribed for which body site? 1Lower eyelids 2External ear canal 3Fingernails and toenails 4Perianal area - Correct answer 1 Unconscious or comatose clients are often unable to close their eyes or do not have a functioning blink reflex. When the eye remains open for a prolonged time, the cornea will dry out, causing irritation or ulceration. The nurse is in the process of inserting a urinary catheter in an adult female client. The nurse advances the catheter approximately 2 to 3 inches (5 to 7 cm), but no urine return is seen. What should the nurse do next? 1Inflate the catheter balloon. 2Advance the catheter a few more inches. 3Withdraw the catheter and try again. 4Notify the health care provider (HCP). - Correct answer 2 A child diagnosed with thalassemia has received several blood transfusions during the past three days. What lab value is the priority for the nurse to monitor with this client? 1Hemoglobin level 2Platelet count 3Blood urea nitrogen level 4Neutrophil percentage - Correct answer 1 A normal hemoglobin range for children is approximately 11 to 13 mg/ld. Thalassemia, also called Cooley's anemia, is a genetic defect that causes anemia, i.e., a condition in which the The nurse is reviewing the chart of a client who was recently diagnosed with coronary artery disease due to atherosclerosis. Which factors most likely contributed to the development of this disease? (Select all that apply.) - Correct answer Mother died of a myocardial infarction Low-density lipoprotein (LDL) level of 149 mg/LD. History of diabetes mellitus Used to smoke 40 packs per year until one year ago The target LDL level for a client is less than 100 mg/LD. The nurse is evaluating a client who was admitted for a small bowel obstruction and dehydration. Which observation by the nurse would indicate that the dehydration is improving? 1The client has normative bowel sounds. 2The client voided 300 mL of urine in the past two hours. 3The client denies any nausea or vomiting. 4The client reports the passing of flatus. - Correct answer 2 A client is admitted to the telemetry unit with syncope due to sinus bradycardia. Which intervention should the nurse include in the client's plan of care? 1Maintain the client on bed rest. 2Administer a stool softener daily. 3Implement seizure precautions. 4Discuss the client's wishes for organ donation. - Correct answer A client is admitted to the telemetry unit with syncope due to sinus bradycardia. Which intervention should the nurse include in the client's plan of care? 1Maintain the client on bed rest. 2Administer a stool softener daily. 3Implement seizure precautions. 4Discuss the client's wishes for organ donation. - Correct answer 2 To avoid a vasovagal response (i.e., the slowing of the heart rate caused by bearing down when trying to defecate) and the risk for another sync opal episode, it is important to ensure that the client's bowel movements are soft and easily expelled. The client should also be instructed to avoid holding their breath or bearing down (Valhalla maneuver). A client diagnosed with iron deficiency anemia is prescribed ferrous sulfate suspension orally. Which instruction would be most appropriate for the nurse to give to the client regarding this medication? 1"You should use a straw when taking this medication." 2'Taking this medication will turn your urine dark orange in color." 3"Diarrhea is a common side effect when taking this medication." 4"You should take the medication with food to enhance absorption." - Correct answer 1 Because liquid iron can stain the teeth, the most appropriate instruction is to use a straw The nurse is caring for a client with severe iron deficiency anemia. Which interventions should the nurse include in the client's plan of care? (Select all that apply.) - Correct answer Instruct assistive personnel to allow the client to rest during care activities. Monitor the client for palpitations and orthostatic hypotension. Review the client's medical record for NSAID use. Encourage the client to eat more green leafy vegetables and beans. Monitor the client's stool for color, consistency and frequency. The nurse is caring for a client with a diagnosis of pericarditis. The unlicensed assistive person reports to the nurse that the client's last set of vital signs were blood pressure of 84/40 mm Hg, respiratory rate of 28 breaths/minute, heart rate of 112 and the client seemed short of breath. The nurse examines the client and also notes the presence of jugular vein distention. What should the nurse do next? 1Administer the prescribed metoprolol. 2Notify the health care provider. 3Place the client on nothing by mouth status. 4Obtain a 12-lead electrocardiogram. - Correct answer 2 Risk for cardiac tamponed due to jugular vein distention The nurse in the outpatient clinic is reviewing the medical record of a client diagnosed with Raynaud's disease. What information from the client's health history would support this diagnosis? (Select all that apply.) - Correct answer the client works in an office setting as a typist. The client smokes two packs of cigarettes per day. Warfarin is listed on the medication reconciliation form. The client complains of brittle fingernails that break easily. Fingers become cyanotic when exposed to cold objects. A client is admitted to the cardiology unit for treatment for recurrent supraventricular tachycardia. Which observation by the nurse would best indicate that the client's condition can be considered hemodynamic ally stable? 1The client denies any chest pain and capillary refill is less than three seconds. 2The client's blood pressure is 88/40 mm Hg. 3The client's pulse dosimeter reads 91% on three liters nasal cannula. 4The client's cardiac monitor shows a heart rate of 170 beats per minute. - Correct answer 1 A client with a history of chronic alcohol use disorder is admitted to the inpatient unit with a serum magnesium level of 1.0 me/L. Which intervention should the nurse implement first? 1Assess the client's deep tendon reflexes. 2Order the client a meal with foods high in magnesium. 3Obtain the client's heart rate and oxygen saturation. 4Place the client on fall risk and seizure precautions. - Correct answer 3 The nurse administered furosemide to a client with acute pulmonary edema. Which observation by the nurse would indicate that the client is experiencing an adverse side effect of the medication? 1The client exhibits exceptional dyspnea with walking. 2The client reports muscle cramps in both legs. 3The client's blood pressure is 104/60 mm Hg. 4The client's weight decreased by 2 lbs. in two days. - Correct answer 2 Muscle cramps and spasms while receiving diuretic therapy could indicate hypokalemia, an adverse drug effect of furosemide because this is a potassium wasting diuretic The nurse is reviewing the plan of care for a client with peripheral artery disease who has a history of leg pain with walking. Which interventions should the nurse include in the client's plan of care? (Select all that apply). - Correct answer Enroll the client in an exercise program that involves low-impact activities. Assist the client in selecting food items that are low in saturated fats and cholesterol. 3Schedule a lung cancer screening for the client. 4Assist the client with enrolling in a smoking cessation program. - Correct answer 4 A nurse is administering the influenza vaccine in an occupational health clinic. Within 10 minutes of giving the vaccine to a middle-aged adult male, the man reports having itchy and watery eyes, feeling anxious and short of breath. What should the nurse do first? 1Administer SQ epinephrine. 2Maintain the airway. 3Take the client's vital signs. 4Apply oxygen. - Correct answer 1 The nurse is evaluating whether teaching a client with dysphagia about preventing aspiration was effective. Which action by the client indicates that additional teaching is required? 1The client is sitting in a chair during meals. 2The client uses a straw to drink. 3The client tucks in the chin while swallowing. 4The client alternates solids with liquids. - Correct answer 2 The nurse is assisting with discharging a client from the hospital who was admitted for acute exacerbation of chronic obstructive pulmonary disease. Which statement by the client indicates that teaching was effective? 1"I will make sure to get the pneumonia vaccine every October." 2"I will eat foods low in calories and protein." 3"I will switch from regular to electronic cigarettes." 4"I will use my spacer each time I use my inhaler." - Correct answer 4 The home health nurse is reviewing information with a client who is being treated for pulmonary tuberculosis. Which statement by the nurse is correct? 1"You should not leave your home until your cough is completely gone." 2"Your family members should get the tuberculosis vaccine." 3"You can stop the medications once your symptoms have resolved." 4"You should avoid public transportation and crowds in enclosed areas." - Correct answer 4 The nurse is caring for a client newly diagnosed with chronic obstructive pulmonary disease (COPD). The nurse reviews the client's medical record and notes which risk factors? (Select all that apply.) - Correct answer it is primarily caused by cigarette smoking. Other risk factors include genetics, asthma and exposure to occupational chemicals and air pollution. ALL HISTORY OF The nurse in the pediatric clinic is caring for an acutely ill, 10-year-old child. Which assessment finding would require immediate intervention by the nurse? 1Slow, irregular respirations 2Temperature of 101.3° F (38.5° C) 3Rapid, bounding pulse 4Profuse diaphoresis - Correct answer 1 The nurse is preparing a client for a pulmonary CT angiogram with contrast to rule out a pulmonary embolism. For which laboratory result should the nurse notify the health care provider immediately? 1D-dimer level of 1.2 mcg/mL 2Serum creatinine level of 2.8 mg/LD. 3Arterial blood gas PaO2 level of 80 mm Hg 4Serum troponin level of 0.1 mg/mL - Correct answer 2 The client's creatinine level is significantly elevated (normal creatinine level is 0.8 to 1.2 mg/ld.), placing the client at risk for dye-induced renal failure and the nurse should notify the health care provider of this lab result immediately. The nurse in the primary health care provider's office is reviewing the medical record of a client with idiopathic pulmonary arterial hypertension. The nurse should expect which potential clinical manifestations with this disease? (Select all that apply.) - Correct answer Classic symptoms include: exceptional dyspnea and chest pain, fatigue, right-sided heart failure (core pulmonale) due to the increased workload of the right ventricle and abnormal heart sounds, such as an S3. The home health nurse is reviewing the medical record of a client with closed-angle glaucoma in both eyes. Which statement by the client would support this diagnosis? 1"I have specks floating in my eyes." 2"I have to turn my head to see around the room." 3"I can't see out of my left eye." 4"I have constant blurred vision." - Correct answer 2 Which action should the nurse take before communicating with a client diagnosed with presbycusis? 1Check the client for crewmen impaction. 2Ask for permission to turn off the television. 3Request a medical translator. 4Wait until family members have left. - Correct answer 2 The nurse is planning care for a client diagnosed with Guillain-Barre syndrome. Which problem should the nurse identify as a priority? 1Difficulty breathing 2Altered bowel elimination 3Partial or total immobility 4Nutritional deficits - Correct answer 1 The nurse is providing care to an 80-year-old client with the diagnosis of advanced Parkinson's disease. The nurse should know that the greatest risk to the client is related to which finding? 1Difficulties with reading and seeing at night 2Extreme weakness in the lower extremities 3Drooling and coughing when eating 4Dizziness and sync opal episodes - Correct answer 3 The home health nurse is reviewing the plan of care for a client experiencing acute attacks of Meniere’s disease. What is the priority intervention for this client? 1Instruct the client not to drive a motor vehicle. 2Provide assistance with bathing and dressing. 3Communicate clearly and use visual aids. 4Encourage bland foods and noncarbonated fluids. - Correct answer 1 The nurse on the inpatient unit is expecting the admission of a client with a new onset of seizures and instructs the unlicensed assistive person (UAP) to prepare the client's room. Which piece of equipment should the UAP make sure to place in the room? Provide assistance with ambulation. The nurse is reviewing the plan of care for a 30-year-old client newly diagnosed with multiple sclerosis. Which interventions should the nurse include for this client? (Select all that apply.) Instruct the client on how to self-catheterize as needed. Review methods to prevent and treat constipation. Encourage participation in physical and occupational therapy. Encourage participation in vocational rehabilitation. Encourage independence in personal care and bathing. - Correct answer Review methods to prevent and treat constipation. Encourage participation in physical and occupational therapy. Encourage participation in vocational rehabilitation. Encourage independence in personal care and bathing. The nurse is caring for a client who has a history of peptic ulcer disease. The nurse notes the abdomen is rigid and the client complains of severe pain with palpation. What is the priority action by the nurse? 1Record the findings in the client's record. 2Ask the client about dietary habits. 3Notify the health care provider of the findings. 4Review the client's record for NSAID use. - Correct answer 3 The nurse is caring for a client who is suffering from an exacerbation of ulcerative colitis. Which manifestations would the nurse expect to see with this client? (Select all that apply.) Fever of 104° F (40° C) Crackles in the lower lung fields Mucous noted in the stool Frequent bloody stools Abdominal pain relieved by defecation - Correct answer mucous, bloody, abdominal pain The nurse is caring for a client who had a small bowel resection two days ago. The client reports that the pain has significantly increased over the last two hours and does not get better after receiving an analgesic. Which additional findings are indicative of a postoperative complication the client might be experiencing? (Select all that apply.) Tenderness at the incision site Taut, distended abdomen Hyperactive bowel sounds Serosanguineous fluid in the surgical drain Nausea and vomiting - Correct answer Taut, distended abdomen Hyperactive bowel sounds Nausea and vomiting The nurse in the primary care office is reviewing after-visit instructions with a client who was recently diagnosed with gastro esophageal reflux disease (GERD). Which action should the client implement to decrease the symptoms associated with GERD? 1Increase oral fluid intake to 4 liters a day. 2Avoid caffeinated and carbonated beverages. 3Eliminate dairy products from the diet. 4Limit foods high in fiber. - Correct answer 2 The home health nurse is caring for a client who underwent a partial gastrostomy due to gastric cancer several months ago. Which finding would indicate that the client is suffering from pernicious anemia? (Select all that apply.) The client's sclera are icteric. The client reports numbness and tingling in the feet. The client is experiencing urinary retention. The client's tongue is shiny and beefy-red. The client is exhibiting alopecia. - Correct answer the client's sclera are icteric. The client reports numbness and tingling in the feet Tongue shiny and beefy-red The nurse is educating a group of individuals about how to prevent hepatitis B and C. Which statement by the nurse would best describe prevention of these two diseases? 1"You can eat fresh fruit picked from the tree without the need to wash." 2"You should use protection when engaging in sexual intercourse." 3"You should talk to your health care provider when traveling internationally." 4"You can receive a yearly vaccination to prevent the diseases." - Correct answer 2 The nurse at the outpatient surgery center is speaking with a client who is scheduled for a colonoscopy the next morning. Which information about the procedure should the nurse make sure to include? (Select all that apply.) "You will have an intravenous catheter inserted prior to the procedure." "You will be required to lay still for 6 to 8 hours after the procedure." "You should only consume clear liquids for the next 12 to 24 hours." "Remember to stop eating any food six hours before you come to the center." "Make sure to drink the entire bowel preparation liquid." - Correct answer "You will have an intravenous catheter inserted prior to the procedure." "You should only consume clear liquids for the next 12 to 24 hours." "Remember to stop eating any food six hours before you come to the center." "Make sure to drink the entire bowel preparation liquid." shrimp, crab and rice. Foods high in phosphorus include: organ meats, salmon, scallops, nuts and cheese. A client comes to the community health clinic with symptoms of gonorrhea. Which intervention should the nurse implement first? 1Discuss the risk of infertility with the client. 2Collect a urethral swab from the client. 3Instruct the client to notify past sexual partners. 4Obtain information about the client's recent sexual encounters. - Correct answer 4 The nurse is assisting with developing a plan of care for a client with benign prostatic hyperplasia. Which nursing interventions should the nurse include for this client? (Select all that apply.) Limit caffeinated and alcoholic beverages. Calculate accurate intake and output. Void every 1 to 2 hours to empty the bladder. Catheterize as needed for post-void residual urine. Monitor for bladder distention. - Correct answer limit caffeine Catheterize as needed Monitor The nurse is reviewing the medical record of a client admitted with acute kidney injury. Which findings would support this diagnosis? (Select all that apply.) Proteinuria Hypokalemia Elevated creatinine level Decreased glomerular filtration rate Hematuria Decreased blood area nitrogen - Correct answer proteinuria Elevated creatinine Decreased function Hematuria A nurse is caring for a client with continuous bladder irrigation (CBI), following a transurethral resection of the prostate. Which finding would indicate the need for the nurse to increase the flow of the CBI? 1Bladder spasms 2Pain at the catheter insertion site 3Temperature of 99.8° F 4Blood clots in the catheter tubing - Correct answer 4 A 68-year-old, postmenopausal, female client has been prescribed tamoxifen for breast cancer with bone metastases. The nurse should reinforce teaching about which potential adverse drug effect? 1Stroke-like symptoms 2Seizures 3Symptoms of hypocalcaemia 4Insomnia - Correct answer 1 Tamoxifen is an antineoplastic drug, commonly prescribed for clients with breast cancer or for clients who are at high risk for developing breast cancer. The most common adverse drug effects (ADEs) are hot flashes, fluid retention, vaginal discharge, nausea, vomiting and menstrual irregularities. In women with bone metastases, tamoxifen may cause transient hyperkalemia. Because of its estrogen agonist actions, tamoxifen poses a small risk of thromboembolic events, including deep vein thrombosis, pulmonary embolism and stroke. The school nurse is teaching a group of teenagers about the prevention of sexually transmitted infections (STIs). Which statement by one of the students indicates an understanding of the teaching? 1"Wearing a condom will eliminate any risk of contracting an STI." 2"There are vaccines available that will prevent the majority of STIs." 3"Being on birth control will prevent getting an STI." 4"Having multiple sexual partners puts me at a higher risk for an STI." - Correct answer 4 The nurse is reinforcing teaching with a client who has recurrent kidney stones. Which statement by the client would indicate that further teaching is needed? 1"I will follow a low-calcium diet and avoid dairy products." 2"I will contact my health care provider if I am having difficulty urinating." 3"I will monitor the color of my urine." 4"I will make sure I drink plenty of water throughout the day." - Correct answer 1 A client with benign prostatic hypertrophy has been prescribed tamsulosin. Which statement by the nurse describes how this medication works? 1"This drug will eliminate your nocturnal." 2"This medication will improve the flow of urine." 3"Your libido will increase with this medication." 4"This medication will shrink your enlarged prostate gland." - Correct answer 2 The nurse in the outpatient clinic is assisting in the admission of a client scheduled for a prostatectomy this morning. Which statement by the client should be of greatest concern to the nurse? 1"I am feeling nervous about the procedure." 2"I have not had anything to eat since 9:00 pm last night." 3"I have had an allergic reaction to an antibiotic before." 4"I have not had to urinate since yesterday evening." - Correct answer 4 The nurse is reviewing the electronic medical record of a client diagnosed with endometriosis. The nurse should expect which findings with this diagnosis? (Select all that apply.) - Correct answer the following findings that would indicate the client has endometriosis are pain with menstruation (dysmenorrhea), pain with intercourse (dyspareunia), excessive bleeding, and infertility. The client may also complain of pelvic and/or back pain, along with pain during bowel movements. The nurse in the emergency department is admitting a client with a reduced level of consciousness due to severe hypothyroidism. Which interventions should the nurse implement first? 1Implement warming blankets as indicated. 2Orient the patient to person, time and place. 3Monitor O2 saturation and provide supplemental oxygen. 4Administer propranolol as prescribed. - Correct answer 3 The nurse is reviewing the plan of care for a client with acute adrenocortical insufficiency. Which intervention should be a priority for this client? 1Administration of potassium supplements 2Electrocardiogram monitoring 3Implementation of a low-sodium diet 4Administration of insulin - Correct answer 2 The nurse understands that the prescribed levothyroxine is effective when the client with hypothyroidism makes which statement? 1"I still feel lethargic and fatigued." 2"I have been having daily, formed bowel movements." 3"I have to change my sheets in the morning because I sweat a lot at night." 4"I was reprimanded at work after becoming angry with my boss." - Correct answer 2 The nurse is caring for a client who presents with polyuria, polydipsia and a urine specific gravity of 1.002. The nurse suspects that the client is experiencing diabetes insipid us. Which risk factors would support this diagnosis? (Select all that apply.) Recent neurologic injury Current use of lithium History of recent surgery History of radiation treatment History of pulmonary disease - Correct answer recent neurologic injury Current use of lithium History of recent surgery History of radiation treatment The nurse is planning care for a client admitted with uncontrolled hyperglycemia. Which activities can the nurse delegate to the unlicensed assistive person (UAP)? (Select all that apply.) Soak the client's feet in warm water prior to performing nail care. Administer insulin, but do not aspirate for blood prior to injecting. Report any skin lesions or breakdown to the nurse. Cut the client's toenails short and trim the corners with cuticle scissors. Apply moisturizing cream between the client's toes. After bathing, ensure that the client's skin is completely dry. Check the client's blood sugar before meals and at bedtime. - Correct answer Report any skin lesions or breakdown to the nurse. After bathing, ensure that the client's skin is completely dry. Check the client's blood sugar before meals and at bedtime. The nurse is caring for a client who has been diagnosed with syndrome of inappropriate antidiuretic hormone (SIADH). Which interventions are appropriate for this client? (Select all that apply.) Monitoring of intake and output Administration of a loop diuretic Implementation of a fluid restriction Implementation of a low-sodium diet Administration of vasopressin - Correct answer monitoring of intake and output Administration of a loop diuretic Implementation of a fluid restriction The nurse is caring for a client who has been diagnosed with Cushing syndrome. Which medication most likely contributed to this condition? Pantoprazole Prednisone Paroxetine Pravastatin - Correct answer prednisone The nurse is reviewing the medical record of a client with diabetes who was admitted for a surgical site infection. Which findings should the nurse report to the health care provider? (Select all that apply.) - Correct answer In reviewing the lab values, the nurse should notify the HCP of the positive glucose in urine (normally, glucose is not seen in urine), A1C of 8% (desired range for a client with diabetes is 7% or less), and the serum glucose level of 220 mg/ld., which is higher than the normal range of 70 to 110 mg/ld. These abnormal lab results indicate that the client's diabetes is not managed well and most likely contributed to the client developing an infection. The nurse is reviewing the medical record of a client who has been diagnosed with osteoporosis. The nurse identifies which risk factors for this condition? (Select all that apply.) The client takes 10 mg of prednisone daily. The client performs weight-bearing exercises six days a week. The client weighs 200 lbs. (90.7 kg) with a height of 5 feet 2 inches (157 cm). The client is a 75-year-old Caucasian female. The client has a 30 pack per year smoking history. - Correct answer Osteoporosis is the loss of bone density that leads to weakness of the bone. Risk factors for osteoporosis include being a postmenopausal woman (lack of estrogen), smoking, thin stature, steroid use, lack of weight- bearing exercise, such as prolonged immobility or a sedentary lifestyle, and ethnicity. Prednisone 75 30pack The nurse in the outpatient clinic is following up on a client with a fractured arm. The client's arm was placed in a cast four hours ago. The client states, "my fingers are tingling and feel cold." Which action should the nurse take first? 1Apply an ice pack to the cast to reduce swelling. 2Elevate the client's arm above the level of the heart. 3Check the capillary refill in the client's fingers. 4Notify the health care provider. - Correct answer 3 A client has received instructions for the management of osteoarthritis. Which statement by the client would indicate a need for additional teaching? 1"Early surgical intervention is the preferred treatment." 2"Gradual weight loss may help my pain." 3"It is important for me to balance my exercise and rest periods." 4"I will avoid driving after I have taken cyclobenzaprine." - Correct answer 1 The nurse is reviewing the medical record of a client who has been diagnosed with systemic lupus erythematous (SLE). The nurse would expect which findings associated with this disease? (Select all that apply.) Generalized weakness Reports of pain in the hands and knees The nurse is reinforcing teaching regarding the use of methotrexate with a female client who has systemic lupus erythematous. Which statement by the client indicates an understanding of the teaching? 1"I should not use contraception that contains estrogen." 2"I will avoid interacting with people in large crowds." 3"Lab work won't be necessary while I take this medication." 4"I will not take any vitamin that contains folic acid." - Correct answer 2 Methotrexate is an immunosuppressant medication that is used to treat systemic lupus erythematous (SLE). Due to immunosuppression, clients taking methotrexate should avoid large crowds of people to prevent becoming ill. Methotrexate should be taken with folic acid to decrease gastrointestinal and hepatic toxicity. Clients who are taking this medication should have a complete blood count test done regularly to monitor for decreased white blood cells and platelets, which can indicate bone marrow suppression. Methotrexate is teratogenicity, therefore, pregnancy should be avoided while taking this medication. The nurse is caring for a client with osteoporosis who has been prescribed alendronate. When providing care, which intervention would be a priority? 1Notify the health care provider if the client reports jaw pain. 2Monitor the client's serum calcium levels. 3Encourage the client to increase their intake of vitamin D. 4Administer the alendronate 30 to 60 minutes before the client eats. - Correct answer 1 The office nurse is discussing how to prevent an acute gouty attack with a client who has gout. Which actions should the nurse recommend to the client? (Select all that apply.) Limit their intake of shellfish and red meats. Take the prescribed prednisone regularly. Limit their consumption of alcohol. Implement stress reduction techniques. - Correct answer limit shellfish/meat intake Limit consumption of alcohol Stress reduction techniques The nurse observes an unlicensed assistive person (UAP) providing care to a client who had a total hip arthroplasty 24 hours ago. Which action by the UAP would require the nurse to intervene immediately? 1Placing non-slip foot wear on the client prior to ambulation. 2Placing a raised toilet seat in the client's bathroom. 3Standing by the client's non-operative side during ambulation. 4Reminding the client not to cross their legs. - Correct answer 3 When assisting the client during ambulation following a total hip arthroplasty, the UAP should stand on the operative side (i.e., the side of the surgery) to help provide support to the client because that is the client's weaker side. The home health care nurse is caring for a client who has epilepsy. While the nurse is providing care, the client has a seizure. Which intervention would be most appropriate to prevent an injury to the client? 1Loosening clothing around the waist 2Asking the client to state where they are 3Lowering the client to the ground 4Placing a pillow under the client's head - Correct answer 3 The nurse is participating in a disaster simulation that involves a school bus accident. The nurse is assigned to care for the following four clients in a rural hospital's emergency department. Which client should the nurse see first? 1The client with a penetrating abdominal wound 2The client with multiple facial abrasions 3The client with an open homers fracture 4The client with a third degree burn to the arm - Correct answer 1 A client with a known large abdominal aortic aneurysm develops a sudden change in level of consciousness and tachycardia. The client's blood pressure is 72/48. What should the nurse do first? 1Activate the hospital's emergency response team. 2Page the client's health care provider. 3Conduct a complete head-to-toe physical assessment. 4Obtain a 12-lead electrocardiogram. - Correct answer 1 The nurse is caring for a client with a medical history of peripheral artery disease, hypertension and smoking. The client reports severe pain in the right lower leg that started very suddenly and did not get better after receiving an analgesic. What action should the nurse take first? 1Check the client's pedal pulse. 2Offer the client an ice pack for the pain. 3Administer an additional dose of the analgesic. 4Notify the health care provider. - Correct answer 1 A client presents to the emergency department with a prolonged asthma attack that did not resolve after the client used a metered-dosed inhaler at home. Which medication should the nurse plan to administer first for this client? 1Oral prednisone 2Fluticasone inhaler 3Intravenous azithromycin 4Nebulized albuterol - Correct answer 4 The nurse on a postpartum nursing unit is receiving report about a client who had a normal spontaneous vaginal delivery the night before. The client has been passing golf ball-sized clots on her peril-pad for the last few hours. The client's most recent blood pressure is 88/56, and her heart rate is 118. The nurse enters the client's room and notices blood oozing from her intravenous insertion site. Which action should the nurse take first? 1Notify the client's health care provider. 2Palpate and massage the client's uterus. 3Perform peril-care and change the client's peril-pad. 4Encourage breastfeeding to promote uterine contractions. - Correct answer 1 The nurse is beginning a shift caring for a group of adult clients on a neurological unit in an acute care hospital. Which client should the nurse see first? 1A client admitted two days ago with an ischemic stroke who has a blood pressure of 158/64 2A client admitted several hours ago with a subdural hematoma due to an unwitnessed fall at home 3A client admitted with hepatic encephalopathy who has an elevated ammonia level
Docsity logo



Copyright © 2024 Ladybird Srl - Via Leonardo da Vinci 16, 10126, Torino, Italy - VAT 10816460017 - All rights reserved